LSAT and Law School Admissions Forum

Get expert LSAT preparation and law school admissions advice from PowerScore Test Preparation.

User avatar
 Dave Killoran
PowerScore Staff
  • PowerScore Staff
  • Posts: 5853
  • Joined: Mar 25, 2011
|
#27196
Complete Question Explanation
(The complete setup for this game can be found here: lsat/viewtopic.php?t=11436)

The correct answer choice is (E)

The initial rules indicate that L can never be tested last. From our discussion of F in the setup explanation, we were also able to deduce that K cannot be last. Ultimately, K and L are the only two variables that cannot be tested last, and therefore F, G, H, I, and M can all be tested last. Answer choice (E) is thus correct.
 ccochran@redw.com
  • Posts: 1
  • Joined: Jul 03, 2021
|
#89235
Isn't this response incorrect, as the only cold medications that could exist when M is chosen is M, F, H, I, and L? L ranks second, so it can't be excluded.
User avatar
 Beatrice Brown
PowerScore Staff
  • PowerScore Staff
  • Posts: 75
  • Joined: Jun 30, 2021
|
#89292
Hi Cochran! Thanks for your question :)

You're correct that when M is chosen, the five chosen medications must be M, F, H, I, and L.

However, this question asks us for a complete list of which variables can be ranked fifth. Since L can never be fifth (the rules state it can only be ranked second), any answer choice that lists L is incorrect. Since answer choice (E) does not list L, it is not incorrect.

I hope this helps, and let me know if you have any further questions!

Get the most out of your LSAT Prep Plus subscription.

Analyze and track your performance with our Testing and Analytics Package.